You are on page 1of 8

EXPLANATIONS FOR Q UIZ I

1. This is a basic percent increase question. The formula you should use is
difference 6 −5 1
× 100 = %change . That works out to be × 100 = × 100 = 20% or
original 5 5
answer choice (D).

2. When you have questions that deal with variables in the exponent, your goal should
always be to convert all your base numbers to the same number, if possible. Dealing with
the left side of the equation first, it should look as follows:
( )
5 25 × 413 = 5 25 × 2 2
13
= 5 25 × 2 26 = (5 × 2) 25 × 2 = 2 ×10 25 . Therefore k = 25 or answer
choice (C).

3. Here, remembering that the difference of two squares can always be rewritten as the
product of the sum and the difference of the two variables is key. It is good to be in the
habit of automatically rewriting the expression m 2 − n 2 as (m + n )(m − n ) . If you do
that, then it is easy to see that the piece of the puzzle you are looking for is the value of
(m + n ) . Statement 2 gives it to you, but statement 1 does not, so the correct answer here
is (B).

4. This is a yes/no data sufficiency question, so the key here is to come up with some
examples that satisfy the parameters you have been given. For statement 1, it is easy to
come up with examples that will give you a "yes" and those that will give you a "no": if
you plug in Set P {1, 3, 5} and Set Q {10, 12, 14} you get a "no", but if you plug in Set P
{11, 13, 15} and Set Q {2, 4, 6}, you will get a "yes". For statement 2, it gets a little bit
tricky, but it is still possible to get both answers: if you plug in Set P {1, 3, 5} and Set Q
{10, 12, 14} you still get a "no", but if you keep Set P the same and change Set Q to {-
100, 12, 14} the median of Q is higher than the mean of P, but Q's mean is less than P's,
so you get a "yes". When you combine the information, however, with consecutive sets
the mean is equal to the median, so you can only come up with "no". Therefore, the
correct answer is (C).

5. This one looks a lot scarier than it actually is. If you know your exponent rules well,
−1
 
  −1 −1
1  1  1
it should be no problem:   =  =   = 13 .
 1 + 1  6+7  13 
 −1 
6 7 −1 

6. This is a question that should be set up with a groups grid:

Women Men Total


Off-campus (1) 120 200
On-campus (2) x
Total 400 (2) ½ x Total
Statement 1 tells you how many on-campus women and how many off-campus men there
are, but not how many total students. From statement 2, you can set up a pair of
1
equations: 400 + x = Total and 200 + x = Total . Whenever you have two equations
2
and two unknowns, you can solve. Therefore, the correct answer is (B).

7. The key to this question, as with many geometry questions, is to draw an additional
line on the diagram. If you draw a line between points A and C, you get an equilateral
triangle. Therefore the correct answer is (C).

8. Start out by finding the total number of possible runner finish arrangements:
6 × 5 × 4 × 3 × 2 × 1 = 720 . Of those arrangements, half of them have No Hope beating
Goldenrod and the other half have Goldenrod winning. Since Goldenrod never comes in
before No Hope, the correct answer is  720 = 360 or (B).
1
2

9. This one is a beast if you do the simplification. On the other hand, it can be solved
quite easily if you ballpark. If you round 80 up to 81, and 5 down to 4, you get:
4
4( 9) + which reduces to something very close to 6, the correct answer. To
9 + 4( 2)
prove it mathematically, the steps you take look like this:
4  1   1 
4 80 + = 4 80 +  = 4 4 5 + 
9+4 5  9+4 5  9+ 4 5 

= 4
( )
 4 5 9 + 4 5 + 1    
 = 4 36 5 + 80 + 1  = 4 × 9 4 5 + 9  = 36 = 6
9+4 5   9+4 5  9+ 4 5 
     

10. As with all yes/no data sufficiency, it is a good idea to get some sense of what kind of
numbers will give you a "yes" and which will give you a "no" before you look at the
m
statements. In this question, mn < when one or the other variable is negative or when
n
0 < n < 1 . All other scenarios will give us a "no". Statement 1 rules out one but not the
other variable being negative, but it does not address the issue of n being a fraction less
than one. Statement 2 does even less, since we do not know if m < 0 . Together, the
statements still do not tell us if 0 < n < 1 , so the correct answer is (E).

11. Here we have a permutations question with some interchangeable elements. You
should start out by finding the number of ways to arrange 7 items: 7!. Then to account
for the interchangeable items, you divide by the number of interchangeable items
7 × 6 × 5 × 4 × 3 × 2 ×1
factorial, so the result looks as follows: . After you reduce the
3 × 2 × 1× 2 × 1× 2 × 1
fraction, you should get 7 × 6 × 5 = 210 or (C).

12. Use the points you can identify to eliminate answer choices. Since the curve passes
through the origin, when x = 0, the result should equal 0 also. By that logic, you can
cross off (B) and (E). Then, in order to have the curve dip below the x-axis to the right of
the y-axis as it does, some positive values of x must result in a negative value for the
expression. The only remaining answer that could ever do that is the correct answer, (D).

13. Statement 1 is certainly not enough information on its own. Nor is statement 2-- it
simply tells you that all of the variables are exponents of 2 (i.e. 2, 4, 8, 16, etc.).
Together, however, you need to slow down and watch out. Is there only one possible
value for d? Yes. First list all exponents of 2 less than 170: 2, 4, 8, 16, 32, 64, 128. Is
there any way to combine four of the numbers to add up to 170 without using 128? If
you add up 8 + 16 + 32 + 64, you will see that even that is too small. Therefore, d must
equal 128 and the correct answer is (C).

7 ×6×5
14. First find the total number of three topping combos that are possible: = 35 .
3 × 2 ×1
Now find out how many of those combine anchovies and pineapple: A, P, and 5 choices
for the third topping makes a total of 5 illegal combinations. So how many legal
combinations are there? 35 - 5 = 30 or (E).

15. Since this is a yes/no type data sufficiency question, take stock of the question before
you dive into the statements. To understand what is really being asked of you, it is
helpful to rearrange the terms in the left side of the inequality to match better those on the
 a  b  c   b  c 
right:     >    . When you do that rearrangement, it becomes clear that
 f  d  e   d  e 
a
the only difference between the two sides is the term. At this point, it becomes clear
f
that statement 1 does nothing to answer the question. For the above inequality to hold
a a
true, > 1 if the right side of the inequality is positive, < 1 if it is negative. To see
f f

how this works, set    = −1 and then 1. Statement 2 ends up telling us very little,
b c
 d  e 
a a
because we can choose values for that result in numbers on either side of 1. ( < 1
f f
when one or both variables is negative). Taking both statements together does little to
resolve any of these issues, so the correct answer is (E).
16. To answer this question you need to find out how many combinations of ingredients
contain the sorbet and then how many combinations of ingredients there are total. If the
dessert contains the sorbet, then there are 8 types of cookies to choose from for the last 2
8×7
ingredients, so you get or 28 combinations. To find all possible combinations, you
2 ×1
9×8×7
have 9 items to choose for three slots: or 84 combinations. The fraction that
3 × 2 ×1
28 1
will contain sorbet then is = or (E).
84 3

17. As with all data sufficiency questions dealing the angles of a triangle, here the key to
the question is remembering that the interior angles of a triangle add up to 180. For
statement 1, you also know that w + z = 180 or z = 180 - w. Plug that into the expression
x + y + z = 180 and you get x + y + 180 - w = 180. Simplify and solve for y, and you get
y = w - x. Therefore, statement 1 is sufficient. To evaluate statement 2, plug 115 in for x
+ z in the equation x + y + z = 180. When you do tha t, you get an equation with only one
variable, which can always be solved. Therefore the correct answer is (D).

18. The ratio of possible five letter passwords to possible four letter passwords is
12 × 11× 10 × 9 × 8
. This reduces to 8 : 1 or (A).
12 × 11 ×10 × 9

19. In statement 1, there are two possible values for )k(: 19 and 28. Only 19 has a
remainder of 3 when divided by 4, however, so statement 1 is sufficient. All even values
of k will have a remainder of 3 when divided by 4 (can you explain why this is so?), so
statement 2 does not give you enough information. Therefore, the correct answer is (A).

20. In this question, it helps to break the question down into steps. First of all, row 1 will
be man, woman, man and row 2 will be woman, man, woman, or vice versa. So we will
deal with only the scenario where row 1 is man, woman, man and row 2 is woman, man,
woman and then double our results. There are three spots in this scenario for three men.
The three men can be arranged in those slots 3 x 2 x 1 = 6 ways. The same for the
women. Multiply the number of arrangements of men by the number of arrangements of
women to get the number of arrangements total for the scenario (6 x 6 = 36). Then, as we
said earlier, double that result and you get 72 or answer choice (A).

21. This question is really asking us if we know for sure whether x and y are positive or
negative. Statement 1 is not sufficient because (x, y) could equal points in all four
quadrants such as (2, 2), (-1, -1), (-1, 0.5), (0.5, -1). Statement 2 is not sufficient because
again, one can find points in all quadrants but III that would work: (10, 10), (-1, 21), (21,
-1). Together, however, it is sufficient, because the only quadrant that now works is I
(points such as (10, 10) or (1, 19)), so the correct answer is (C).
22. The fact that the 6 people are sitting around a table turns out not to matter in this
particular question (although it will in other questions). Rewrite the question so that you
are looking for how many ways to arrange the people in a straight line so that you place
the three democrats first and then the three republicans. This is just 3 x 2 x 1 x 3 x 2 x 1
or 36. Therefore the correct answer is (B).

23. Statement 1 can be expressed as 100E + 0.5 (20C) = 520 (where E = number of $100
checks and C = number of $20 checks). There are 2 variables and only one equation, so
on its own, statement 1 is not sufficient. Statement 2 can be expressed as 100E + 20C =
740. This also is not sufficient on its own. Together, however, the statements are
sufficient, since you have two equations and two unknowns.

24. First find the probability that it will rain on one arrangement of two rainy days and
three sunny days (i.e. it will rain the first two days and then not rain for the last three
4 4 6 6 6 2 2 33
days). This works out as follows: × × × × = 5 . Now find how many
10 10 10 10 10 5
5 × 4 × 3 × 2 ×1
ways there are to arrange 2 rainy days and 3 sunny days: = 10. Now
2 × 1× 3 × 2 × 1
multiply the probability of any one arrangement happening times the number of
2 23 3 23 3 3
arrangements and you get × 10 = or answer choice (E).
55 54

25. This is a permutations question where there are some interchangeable elements, so
you find the total number of arrangements and then divide by the number of
7!
interchangeable elements factorial: = 7 × 6 × 5 × 4 = 840 or (D).
3!

26. In general terms the area of the portion you are looking for can be described as
triangle − circle
. The area of the triangle is fairly straight forward: the base equals 6 and
3
the height equals 3 3 , so the area equals 9 3 . In order to find the area of the circle, we
need to find the radius, and this is a bit tricky, but if you draw the right lines on your
diagram, it is doable. First draw lines bisecting each of the three angles in the triangle.
This forms six smaller 30:60:90 triangles. The shortest side of each of those triangles is
equal to the radius of the circle. We know that the middle length side is half of the side
of the equilateral, so it is equal to 3. Plug that into the side length ratio of the 30:60:90
triangle (1: 3 : 2) so the side lengths here are 3 : 3 : 2 3 . This means that the radius on
the circle is 3 , and the area of the circle is 3π . Plug all of this information into our
initial equation and we get the area of the portion described by the minor arc DE and the
9 3 − 3π
line segments AD and AE is equal to = 3 3 − π or answer choice (B).
3
27. This is a very straightforward probability question. The probability of ge tting an
orange and then a purple stone is    =
4 9 3
. The correct answer is (E).
 16  15  20

28. This one is hard to sort out at first, but if you plug in for a and b as specific values of
x, the question begins to make more sense. For example, if a = 2, b = 3, and a + b = 5,
then the answer choices respond as follows:

( ) (
(A) 5 2 + 1 ≠ 2 2 + 1 + 3 2 + 1 )
(
(B) 2 5 ≠ 2 2 + 2 3 ) ( )
(C) 5 − 4 ≠ (2 − 4) + (3 − 4 )

5  5   5
(D) ≠  + 
5  2   3

(E) − 7(5) = −7(2 ) + (− 7 )(3)

Therefore the correct answer is (E).

29. This one is tough because there are a number of different steps you have to take to get
to the answer. This is a combinations question with two wrinkles: we don't know exactly
how many man and women are serving on the committee and we have some illegal
combinations. There are two ways to distribute the committee slots between men and
women: 3 of each or 4 men and 2 women. We must treat the two scenarios separately. If
there are 3 of each sex on the committee, then the total number of possible combinations
5 × 4 × 3× 8 × 7 × 6
given this scenario is found as follows: = 560 . Of those combinations,
3 × 2 × 1× 3 × 2 × 1
5 × 4 × 3× 6
= 60 (ways of grouping the three women and one man placed on the
3 × 2 × 1× 1
committee with the two men who won't serve together) have the forbidden two men
serving together. Therefore, in this scenario, there are 500 legal combinations. In the
other scenario, where 4 men and 2 women are serving, the total number of combinations
5 × 4 × 3× 2 ×8 × 7 5 × 4 × 3× 2
are = 140 . Of these, = 5 are illegal. Therefore in this
4 × 3 × 2 × 1× 2 × 1 4 × 3× 2 ×1
scenario there are a total of 135 legal scenarios. This means that 635 different
committees of 6 people with at least 3 men and 2 women on the committee are possible,
given that two of the men won't serve together. The correct answer is (E).
30. This function should remind you of factorials, and indeed, the expression [22] + [24]
can be rewritten as 2 10 (10!) + 211 (11!) . (Note that the function is defined as the product of
all even numbers less than x, not less than or equal to x.) The common factors of the two
terms can be factored out, so the expression is rewritten as
( ) ( )
2 10 (10!)(1 + 2(11)) = 210 (10!)( 23) . The largest prime factor of any of these three factors
is 23. Therefore, the correct answer is A.

# favorable
31. As with all probability questions, the equation you are us ing is . Here
# total
you should use your knowledge of permutations to find the elements of the formula.
There are 6! ways that one person is going to each floor, and each of 6 people have 6
choices as to which button to push, which means that there are 66 total ways people's
destinations are distributed. Therefore, the correct answer is (A).

32. Here, plugging in for n is key. If n = 5, then you have the following expression:

r6 − r5 =
( − 1)5 1
= − . This means that r5 is larger than r6 by 1/5. Eliminate answer
5 5
choices (A), (C), and (E). Now plug in n = 6 to see how r7 relates to everything. When

you do, you get r7 − r6 =


( − 1)6
1
=
. This means that r7 is larger than r6 but less than r5 .
6 6
Therefore the correct answer is (D).

33. When you see the words "at least" in a probability question, chances are it is easier to
find the probability of the opposite happening and then subtract from 1. The opposite of
at least one ball being black is both balls being white. The probability of drawing two
7 6 7
white balls is × = . Don't forget to subtract that from 1! So the correct answer is
10 9 15
(D).

34. This looks scary, but if you factor out your common factors from each term, it gets
( )
easier: 2 74 − 2 70 = 2 70 2 4 − 1 = 2 70 (15 ) , so the largest prime factor is 5.

35. The total number of possible pin numbers are 10 x 10 x 10 = 1000. If all but 327 of
them are in use, that means that there are 673 club members. The correct answer is (B).

36. This one is all about exponents:


0.08 −5 0.04 4 (4 × 10 −2 ) (2 2 ) × 10 −8 = 28−15 × 10 − 8− (−10) = 10 2 = 25 or answer
4 4

= = =
0.04 −4 0.08 5 (8 × 10 −2 )5 (2 3 )5 × 10 −10 27 32
choice (C).
37. This question is similar to question 24, only even harder. Since the probability of rain
is 50%, any arrangement of rain and no rain is going to have the same probability of
1 1 1 1 1 1 1
occurring: × × × × × = 6 . So then we have to find how many different
2 2 2 2 2 2 2
arrangements will satisfy our conditions. Even though the question uses "at least", it does
us no good to find the opposite and subtract from 1 because the opposite is just as hard to
find. So we have to go step by step using permutations instead. If it rains on exactly 3
6 × 5× 4 × 3× 2 ×1
out of 6 days, there are = 20 ways that could happen. If it rains on 4
3 × 2 × 1× 3 × 2 × 1
6 × 5 × 4 × 3 × 2 ×1
out of 6 days, there are = 15 ways that could happen. If it rains on 5
4 × 3 × 2 ×1 × 2 ×1
6 × 5× 4 × 3× 2 ×1
out of 6 days, there are = 6 ways that could happen. If it rains all 6
5 × 4 × 3 × 2 × 1 ×1
days, there is, of course, only 1 way for that to happen. Therefore, for it to rain on at
least 3 days, there are 20 + 15 + 6 + 1 = 42 ways that could happen. So the probability
42 21
that it will rain on at least 3 days is 6 = 5 , or answer choice (D).
2 2

You might also like